Linear Algebra (Change of Basis)

Click For Summary
SUMMARY

The discussion focuses on proving that the set G={1+x, 1-x, 1-x², 1-x³} forms a basis for the polynomial space P3, where E={1, x, x², x³} is the standard basis. The change of basis matrix S from G to E is correctly identified as S_E^G=\left(\begin{array}{cccc} 1 & 1 & 1 & 1 \\ 1 & -1 & 0 & 0 \\ 0 & 0 & -1 & 0 \\ 0 & 0 & 0 & -1 \end{array}\right). To confirm G as a basis, it is essential to demonstrate that G is linearly independent and spans P3, given that P3 has a dimension of 4.

PREREQUISITES
  • Understanding of polynomial spaces, specifically P3.
  • Knowledge of linear independence and spanning sets in vector spaces.
  • Familiarity with change of basis matrices in linear algebra.
  • Ability to perform matrix multiplication and interpret results in the context of vector spaces.
NEXT STEPS
  • Learn how to determine linear independence of a set of vectors in polynomial spaces.
  • Study the properties of change of basis matrices in linear algebra.
  • Explore the concept of spanning sets and their significance in vector spaces.
  • Practice problems involving transformations between different bases in polynomial spaces.
USEFUL FOR

Students and educators in linear algebra, mathematicians working with polynomial spaces, and anyone interested in understanding the concepts of basis and change of basis in vector spaces.

DanielFaraday
Messages
86
Reaction score
0

Homework Statement



Let E={1, x, x2,x3} be the standard ordered basis for the space P3. Show that G={1+x,1-x,1-x2,1-x3} is also a basis for P3, and write the change of basis matrix S from G to E.

Homework Equations

The Attempt at a Solution



Here's what I got:

<br /> S_E^G=\left(<br /> \begin{array}{cccc}<br /> 1 &amp; 1 &amp; 1 &amp; 1 \\<br /> 1 &amp; -1 &amp; 0 &amp; 0 \\<br /> 0 &amp; 0 &amp; -1 &amp; 0 \\<br /> 0 &amp; 0 &amp; 0 &amp; -1<br /> \end{array}<br /> \right)<br />

Now, to prove that this is also a basis, I just need to show that it has an inverse, right?

Here's the problem. If the above is correct, then when you multiply it by G you should get E, right? After all, it is the "change of basis matrix S from G to E". However, this isn't the case:

<br /> \left(<br /> \begin{array}{cccc}<br /> 1 &amp; 1 &amp; 1 &amp; 1 \\<br /> 1 &amp; -1 &amp; 0 &amp; 0 \\<br /> 0 &amp; 0 &amp; -1 &amp; 0 \\<br /> 0 &amp; 0 &amp; 0 &amp; -1<br /> \end{array}<br /> \right).\left(<br /> \begin{array}{c}<br /> 1+x \\<br /> 1-x \\<br /> 1-x^2 \\<br /> 1-x^3<br /> \end{array}<br /> \right)=\left(<br /> \begin{array}{c}<br /> 4-x^2-x^3 \\<br /> 2 x \\<br /> -1+x^2 \\<br /> -1+x^3<br /> \end{array}<br /> \right)<br />

Am I doing something wrong, or am I just confused about what a change of basis matrix is supposed to do?
 
Physics news on Phys.org
Your vectors in your representation in R4 are going to have just numbers, not polynomials in their entries. So (1+x, 1-x, 1-x2, 1-x3) isn't a vector, instead the vectors are supposed to be representing polynomials, for example 1-x is represented as (1,-1,0,0).

Your change of basis matrix is correct. Don't use it to show G is a basis though... you know P3 has dimension 4, so show G is linearly independent and hence since it has 4 elements it must span the space also
 
I see. so a valid test would be:

<br /> \left(<br /> \begin{array}{cccc}<br /> 1 &amp; 1 &amp; 1 &amp; 1 \\<br /> 1 &amp; -1 &amp; 0 &amp; 0 \\<br /> 0 &amp; 0 &amp; -1 &amp; 0 \\<br /> 0 &amp; 0 &amp; 0 &amp; -1<br /> \end{array}<br /> \right).\left(<br /> \begin{array}{c}<br /> 0 \\<br /> 0 \\<br /> 1 \\<br /> 0<br /> \end{array}<br /> \right)=\left(<br /> \begin{array}{c}<br /> 1 \\<br /> 0 \\<br /> -1 \\<br /> 0<br /> \end{array}<br /> \right)<br />

And the fact that I get (1,0,-1,0) says that I have 1-x^2, which is what I would expect.

Cool, I think I got it. Thanks!
 
Don't use it to show G is a basis though... you know P3 has dimension 4, so show G is linearly independent and hence since it has 4 elements it must span the space also

Although that is quite valid, in this problem I would find it simpler to show that G spans P3 and thus is linearly independent.
 
Question: A clock's minute hand has length 4 and its hour hand has length 3. What is the distance between the tips at the moment when it is increasing most rapidly?(Putnam Exam Question) Answer: Making assumption that both the hands moves at constant angular velocities, the answer is ## \sqrt{7} .## But don't you think this assumption is somewhat doubtful and wrong?

Similar threads

  • · Replies 5 ·
Replies
5
Views
2K
  • · Replies 2 ·
Replies
2
Views
1K
  • · Replies 3 ·
Replies
3
Views
2K
  • · Replies 4 ·
Replies
4
Views
1K
  • · Replies 11 ·
Replies
11
Views
3K
  • · Replies 2 ·
Replies
2
Views
2K
  • · Replies 14 ·
Replies
14
Views
3K
Replies
6
Views
1K
Replies
1
Views
2K
  • · Replies 9 ·
Replies
9
Views
2K